Suppose $f$ is defined and differentiable proof. Announcing the arrival of Valued Associate #679: Cesar Manara Planned maintenance scheduled April 23, 2019 at 00:00UTC (8:00pm US/Eastern)Derivatives and continuity of one variable functionsSuppose that $f: mathbb R^q to mathbb R^p$ is a linear map. Prove that $f$ is differentiable and that $f'(x) = f$ for every $x in mathbb R^q$Propositions concerning a differentiable function.Differentiable function and limitProve that $f(x)$ is defined and differentiable for $x>0$Suppose $f$ is differentiable at zero and $f(0) = 0$. Show that $f(x) = xg(x)$Calculating limit of the form of 1×inf - infSuppose $f(x)$ is differentiable on $[0,1]$, and $f(0)=0$, $f(x)ne 0,forall xin(0,1)$Is the piecewise-defined function differentiablebounded differentiable functions

Product of Mrówka space and one point compactification discrete space.

What is "gratricide"?

Is there a kind of relay that only consumes power when switching?

How do living politicians protect their readily obtainable signatures from misuse?

How to unroll a parameter pack from right to left

Why do we bend a book to keep it straight?

If Windows 7 doesn't support WSL, then what does Linux subsystem option mean?

How do I find out the mythology and history of my Fortress?

Do wooden building fires get hotter than 600°C?

Putting class ranking in CV, but against dept guidelines

What is the meaning of 'breadth' in breadth first search?

How to dry out epoxy resin faster than usual?

What initially awakened the Balrog?

Belief In God or Knowledge Of God. Which is better?

How to draw/optimize this graph with tikz

Why weren't discrete x86 CPUs ever used in game hardware?

Parallel Computing Problem

Strange behavior of Object.defineProperty() in JavaScript

Sentence order: Where to put もう

Can anything be seen from the center of the Boötes void? How dark would it be?

How does the math work when buying airline miles?

Is it fair for a professor to grade us on the possession of past papers?

MLE of the unknown radius

What is the appropriate index architecture when forced to implement IsDeleted (soft deletes)?



Suppose $f$ is defined and differentiable proof.



Announcing the arrival of Valued Associate #679: Cesar Manara
Planned maintenance scheduled April 23, 2019 at 00:00UTC (8:00pm US/Eastern)Derivatives and continuity of one variable functionsSuppose that $f: mathbb R^q to mathbb R^p$ is a linear map. Prove that $f$ is differentiable and that $f'(x) = f$ for every $x in mathbb R^q$Propositions concerning a differentiable function.Differentiable function and limitProve that $f(x)$ is defined and differentiable for $x>0$Suppose $f$ is differentiable at zero and $f(0) = 0$. Show that $f(x) = xg(x)$Calculating limit of the form of 1×inf - infSuppose $f(x)$ is differentiable on $[0,1]$, and $f(0)=0$, $f(x)ne 0,forall xin(0,1)$Is the piecewise-defined function differentiablebounded differentiable functions










0












$begingroup$


Suppose $f$ is defined and differentiable for every $x>0$, and $lim_xto infty f'(x)=0$.

Put $g(x)=f(x+1)-f(x)$. Prove that $lim_xto infty g(x)=0$










share|cite|improve this question











$endgroup$











  • $begingroup$
    To get focused answers, it is advisable to indicate effort in your question: what have you tried? Where are you stuck? What are you unsure about? Do you know the definitions of all the concepts involved?
    $endgroup$
    – avs
    Mar 27 at 20:15










  • $begingroup$
    You keep adding new information in the comments about what your professor said to do or not to do. This information should really be in the question: otherwise, people keep trying to answer with incomplete information.
    $endgroup$
    – avs
    Mar 28 at 5:54















0












$begingroup$


Suppose $f$ is defined and differentiable for every $x>0$, and $lim_xto infty f'(x)=0$.

Put $g(x)=f(x+1)-f(x)$. Prove that $lim_xto infty g(x)=0$










share|cite|improve this question











$endgroup$











  • $begingroup$
    To get focused answers, it is advisable to indicate effort in your question: what have you tried? Where are you stuck? What are you unsure about? Do you know the definitions of all the concepts involved?
    $endgroup$
    – avs
    Mar 27 at 20:15










  • $begingroup$
    You keep adding new information in the comments about what your professor said to do or not to do. This information should really be in the question: otherwise, people keep trying to answer with incomplete information.
    $endgroup$
    – avs
    Mar 28 at 5:54













0












0








0





$begingroup$


Suppose $f$ is defined and differentiable for every $x>0$, and $lim_xto infty f'(x)=0$.

Put $g(x)=f(x+1)-f(x)$. Prove that $lim_xto infty g(x)=0$










share|cite|improve this question











$endgroup$




Suppose $f$ is defined and differentiable for every $x>0$, and $lim_xto infty f'(x)=0$.

Put $g(x)=f(x+1)-f(x)$. Prove that $lim_xto infty g(x)=0$







calculus analysis






share|cite|improve this question















share|cite|improve this question













share|cite|improve this question




share|cite|improve this question








edited Mar 27 at 20:31









Bernard

124k742117




124k742117










asked Mar 27 at 20:11









BernhardRiemannBernhardRiemann

12




12











  • $begingroup$
    To get focused answers, it is advisable to indicate effort in your question: what have you tried? Where are you stuck? What are you unsure about? Do you know the definitions of all the concepts involved?
    $endgroup$
    – avs
    Mar 27 at 20:15










  • $begingroup$
    You keep adding new information in the comments about what your professor said to do or not to do. This information should really be in the question: otherwise, people keep trying to answer with incomplete information.
    $endgroup$
    – avs
    Mar 28 at 5:54
















  • $begingroup$
    To get focused answers, it is advisable to indicate effort in your question: what have you tried? Where are you stuck? What are you unsure about? Do you know the definitions of all the concepts involved?
    $endgroup$
    – avs
    Mar 27 at 20:15










  • $begingroup$
    You keep adding new information in the comments about what your professor said to do or not to do. This information should really be in the question: otherwise, people keep trying to answer with incomplete information.
    $endgroup$
    – avs
    Mar 28 at 5:54















$begingroup$
To get focused answers, it is advisable to indicate effort in your question: what have you tried? Where are you stuck? What are you unsure about? Do you know the definitions of all the concepts involved?
$endgroup$
– avs
Mar 27 at 20:15




$begingroup$
To get focused answers, it is advisable to indicate effort in your question: what have you tried? Where are you stuck? What are you unsure about? Do you know the definitions of all the concepts involved?
$endgroup$
– avs
Mar 27 at 20:15












$begingroup$
You keep adding new information in the comments about what your professor said to do or not to do. This information should really be in the question: otherwise, people keep trying to answer with incomplete information.
$endgroup$
– avs
Mar 28 at 5:54




$begingroup$
You keep adding new information in the comments about what your professor said to do or not to do. This information should really be in the question: otherwise, people keep trying to answer with incomplete information.
$endgroup$
– avs
Mar 28 at 5:54










5 Answers
5






active

oldest

votes


















2












$begingroup$

Hint:
$$
f(x + 1) - f(x) = int_x^x+1 f'(s) ; ds,
$$

and $f'(s)$ is "getting smaller" as $s$ increases.






share|cite|improve this answer









$endgroup$












  • $begingroup$
    i am confused because our professor said the derivative use the definition
    $endgroup$
    – BernhardRiemann
    Mar 27 at 20:20










  • $begingroup$
    sorry, not understanding your comment.
    $endgroup$
    – avs
    Mar 27 at 21:33










  • $begingroup$
    For each positive $epsilon$, there exists an $M_epsilon$ such that $$ |f'(s)| < epsilon quad mbox for s > M_epsilon. $$ Therefore, for $x > M_epsilon$, $$ |f(x+1) - f(x)| = left| int_x^x+1 f'(s) ds right| leq int_x^x+1 |f'(s)| ds leq int_x^x+1 ; epsilon ; ds. $$
    $endgroup$
    – avs
    Mar 28 at 16:19



















1












$begingroup$

Need to show: given any positive epsilon, there's a positive M such that |g(x)| is less than epsilon for x beyond M.



So, take an arbitrary positive epsilon.
By the same limit definition, there exists an "M" such that |f'(x)| is small for x beyond M.



Using the mean-value theorem I think you can show that the same M works for |g(x)| too.






share|cite|improve this answer









$endgroup$












  • $begingroup$
    Yes , our professor mentioned about exactly but I did not understand anything
    $endgroup$
    – BernhardRiemann
    Mar 27 at 21:23


















1












$begingroup$

By the mean value theorem for differentiable functions, for $x>0$ there exists $x< c_x< x+1$ such that $g(x)=f(x+1)-f(x)=f'(c_x)$.



If $xrightarrow infty$, then $c_x rightarrow infty$. Therefore,
$lim_xrightarrow inftyg(x)=lim_xrightarrowinftyf'(c_x)=lim_xrightarrowinftyf'(x)=0$






share|cite|improve this answer









$endgroup$




















    1












    $begingroup$

    MVT:



    $g(x)=f(x+1)-f(x)= f'(t_x) cdot 1$; where $x <t_x <x+1$.



    We have $lim_x rightarrow infty f'(x)=0:$



    Let $epsilon >0$ be given.



    There is a $M >0$, real, s.t. for $x >M$:



    $|f'(x)| < epsilon.$



    For $x >M$:



    Since $g(x)=f'(t_x)$ , where $x <t_x,$ we have



    $|g(x)| =|f'(t_x)| < epsilon,$ i .e



    $lim_x rightarrow inftyg(x)=0.$






    share|cite|improve this answer











    $endgroup$




















      0












      $begingroup$

      The proof is simple:



      $lim_x rightarrow infty g(x)=lim_xrightarrow infty f(x+1)-lim_xrightarrow infty f(x)$



      For large $x$, $x+1 approx x$, so, by the previous result,



      $lim_x rightarrow infty g(x)=0$






      share|cite|improve this answer









      $endgroup$












      • $begingroup$
        One has to also prove that $lim_xrightarrow infty f(x)$ exists!
        $endgroup$
        – folouer of kaklas
        Mar 27 at 21:30












      Your Answer








      StackExchange.ready(function()
      var channelOptions =
      tags: "".split(" "),
      id: "69"
      ;
      initTagRenderer("".split(" "), "".split(" "), channelOptions);

      StackExchange.using("externalEditor", function()
      // Have to fire editor after snippets, if snippets enabled
      if (StackExchange.settings.snippets.snippetsEnabled)
      StackExchange.using("snippets", function()
      createEditor();
      );

      else
      createEditor();

      );

      function createEditor()
      StackExchange.prepareEditor(
      heartbeatType: 'answer',
      autoActivateHeartbeat: false,
      convertImagesToLinks: true,
      noModals: true,
      showLowRepImageUploadWarning: true,
      reputationToPostImages: 10,
      bindNavPrevention: true,
      postfix: "",
      imageUploader:
      brandingHtml: "Powered by u003ca class="icon-imgur-white" href="https://imgur.com/"u003eu003c/au003e",
      contentPolicyHtml: "User contributions licensed under u003ca href="https://creativecommons.org/licenses/by-sa/3.0/"u003ecc by-sa 3.0 with attribution requiredu003c/au003e u003ca href="https://stackoverflow.com/legal/content-policy"u003e(content policy)u003c/au003e",
      allowUrls: true
      ,
      noCode: true, onDemand: true,
      discardSelector: ".discard-answer"
      ,immediatelyShowMarkdownHelp:true
      );



      );













      draft saved

      draft discarded


















      StackExchange.ready(
      function ()
      StackExchange.openid.initPostLogin('.new-post-login', 'https%3a%2f%2fmath.stackexchange.com%2fquestions%2f3165068%2fsuppose-f-is-defined-and-differentiable-proof%23new-answer', 'question_page');

      );

      Post as a guest















      Required, but never shown

























      5 Answers
      5






      active

      oldest

      votes








      5 Answers
      5






      active

      oldest

      votes









      active

      oldest

      votes






      active

      oldest

      votes









      2












      $begingroup$

      Hint:
      $$
      f(x + 1) - f(x) = int_x^x+1 f'(s) ; ds,
      $$

      and $f'(s)$ is "getting smaller" as $s$ increases.






      share|cite|improve this answer









      $endgroup$












      • $begingroup$
        i am confused because our professor said the derivative use the definition
        $endgroup$
        – BernhardRiemann
        Mar 27 at 20:20










      • $begingroup$
        sorry, not understanding your comment.
        $endgroup$
        – avs
        Mar 27 at 21:33










      • $begingroup$
        For each positive $epsilon$, there exists an $M_epsilon$ such that $$ |f'(s)| < epsilon quad mbox for s > M_epsilon. $$ Therefore, for $x > M_epsilon$, $$ |f(x+1) - f(x)| = left| int_x^x+1 f'(s) ds right| leq int_x^x+1 |f'(s)| ds leq int_x^x+1 ; epsilon ; ds. $$
        $endgroup$
        – avs
        Mar 28 at 16:19
















      2












      $begingroup$

      Hint:
      $$
      f(x + 1) - f(x) = int_x^x+1 f'(s) ; ds,
      $$

      and $f'(s)$ is "getting smaller" as $s$ increases.






      share|cite|improve this answer









      $endgroup$












      • $begingroup$
        i am confused because our professor said the derivative use the definition
        $endgroup$
        – BernhardRiemann
        Mar 27 at 20:20










      • $begingroup$
        sorry, not understanding your comment.
        $endgroup$
        – avs
        Mar 27 at 21:33










      • $begingroup$
        For each positive $epsilon$, there exists an $M_epsilon$ such that $$ |f'(s)| < epsilon quad mbox for s > M_epsilon. $$ Therefore, for $x > M_epsilon$, $$ |f(x+1) - f(x)| = left| int_x^x+1 f'(s) ds right| leq int_x^x+1 |f'(s)| ds leq int_x^x+1 ; epsilon ; ds. $$
        $endgroup$
        – avs
        Mar 28 at 16:19














      2












      2








      2





      $begingroup$

      Hint:
      $$
      f(x + 1) - f(x) = int_x^x+1 f'(s) ; ds,
      $$

      and $f'(s)$ is "getting smaller" as $s$ increases.






      share|cite|improve this answer









      $endgroup$



      Hint:
      $$
      f(x + 1) - f(x) = int_x^x+1 f'(s) ; ds,
      $$

      and $f'(s)$ is "getting smaller" as $s$ increases.







      share|cite|improve this answer












      share|cite|improve this answer



      share|cite|improve this answer










      answered Mar 27 at 20:16









      avsavs

      4,280515




      4,280515











      • $begingroup$
        i am confused because our professor said the derivative use the definition
        $endgroup$
        – BernhardRiemann
        Mar 27 at 20:20










      • $begingroup$
        sorry, not understanding your comment.
        $endgroup$
        – avs
        Mar 27 at 21:33










      • $begingroup$
        For each positive $epsilon$, there exists an $M_epsilon$ such that $$ |f'(s)| < epsilon quad mbox for s > M_epsilon. $$ Therefore, for $x > M_epsilon$, $$ |f(x+1) - f(x)| = left| int_x^x+1 f'(s) ds right| leq int_x^x+1 |f'(s)| ds leq int_x^x+1 ; epsilon ; ds. $$
        $endgroup$
        – avs
        Mar 28 at 16:19

















      • $begingroup$
        i am confused because our professor said the derivative use the definition
        $endgroup$
        – BernhardRiemann
        Mar 27 at 20:20










      • $begingroup$
        sorry, not understanding your comment.
        $endgroup$
        – avs
        Mar 27 at 21:33










      • $begingroup$
        For each positive $epsilon$, there exists an $M_epsilon$ such that $$ |f'(s)| < epsilon quad mbox for s > M_epsilon. $$ Therefore, for $x > M_epsilon$, $$ |f(x+1) - f(x)| = left| int_x^x+1 f'(s) ds right| leq int_x^x+1 |f'(s)| ds leq int_x^x+1 ; epsilon ; ds. $$
        $endgroup$
        – avs
        Mar 28 at 16:19
















      $begingroup$
      i am confused because our professor said the derivative use the definition
      $endgroup$
      – BernhardRiemann
      Mar 27 at 20:20




      $begingroup$
      i am confused because our professor said the derivative use the definition
      $endgroup$
      – BernhardRiemann
      Mar 27 at 20:20












      $begingroup$
      sorry, not understanding your comment.
      $endgroup$
      – avs
      Mar 27 at 21:33




      $begingroup$
      sorry, not understanding your comment.
      $endgroup$
      – avs
      Mar 27 at 21:33












      $begingroup$
      For each positive $epsilon$, there exists an $M_epsilon$ such that $$ |f'(s)| < epsilon quad mbox for s > M_epsilon. $$ Therefore, for $x > M_epsilon$, $$ |f(x+1) - f(x)| = left| int_x^x+1 f'(s) ds right| leq int_x^x+1 |f'(s)| ds leq int_x^x+1 ; epsilon ; ds. $$
      $endgroup$
      – avs
      Mar 28 at 16:19





      $begingroup$
      For each positive $epsilon$, there exists an $M_epsilon$ such that $$ |f'(s)| < epsilon quad mbox for s > M_epsilon. $$ Therefore, for $x > M_epsilon$, $$ |f(x+1) - f(x)| = left| int_x^x+1 f'(s) ds right| leq int_x^x+1 |f'(s)| ds leq int_x^x+1 ; epsilon ; ds. $$
      $endgroup$
      – avs
      Mar 28 at 16:19












      1












      $begingroup$

      Need to show: given any positive epsilon, there's a positive M such that |g(x)| is less than epsilon for x beyond M.



      So, take an arbitrary positive epsilon.
      By the same limit definition, there exists an "M" such that |f'(x)| is small for x beyond M.



      Using the mean-value theorem I think you can show that the same M works for |g(x)| too.






      share|cite|improve this answer









      $endgroup$












      • $begingroup$
        Yes , our professor mentioned about exactly but I did not understand anything
        $endgroup$
        – BernhardRiemann
        Mar 27 at 21:23















      1












      $begingroup$

      Need to show: given any positive epsilon, there's a positive M such that |g(x)| is less than epsilon for x beyond M.



      So, take an arbitrary positive epsilon.
      By the same limit definition, there exists an "M" such that |f'(x)| is small for x beyond M.



      Using the mean-value theorem I think you can show that the same M works for |g(x)| too.






      share|cite|improve this answer









      $endgroup$












      • $begingroup$
        Yes , our professor mentioned about exactly but I did not understand anything
        $endgroup$
        – BernhardRiemann
        Mar 27 at 21:23













      1












      1








      1





      $begingroup$

      Need to show: given any positive epsilon, there's a positive M such that |g(x)| is less than epsilon for x beyond M.



      So, take an arbitrary positive epsilon.
      By the same limit definition, there exists an "M" such that |f'(x)| is small for x beyond M.



      Using the mean-value theorem I think you can show that the same M works for |g(x)| too.






      share|cite|improve this answer









      $endgroup$



      Need to show: given any positive epsilon, there's a positive M such that |g(x)| is less than epsilon for x beyond M.



      So, take an arbitrary positive epsilon.
      By the same limit definition, there exists an "M" such that |f'(x)| is small for x beyond M.



      Using the mean-value theorem I think you can show that the same M works for |g(x)| too.







      share|cite|improve this answer












      share|cite|improve this answer



      share|cite|improve this answer










      answered Mar 27 at 20:47









      bluebirdbluebird

      214




      214











      • $begingroup$
        Yes , our professor mentioned about exactly but I did not understand anything
        $endgroup$
        – BernhardRiemann
        Mar 27 at 21:23
















      • $begingroup$
        Yes , our professor mentioned about exactly but I did not understand anything
        $endgroup$
        – BernhardRiemann
        Mar 27 at 21:23















      $begingroup$
      Yes , our professor mentioned about exactly but I did not understand anything
      $endgroup$
      – BernhardRiemann
      Mar 27 at 21:23




      $begingroup$
      Yes , our professor mentioned about exactly but I did not understand anything
      $endgroup$
      – BernhardRiemann
      Mar 27 at 21:23











      1












      $begingroup$

      By the mean value theorem for differentiable functions, for $x>0$ there exists $x< c_x< x+1$ such that $g(x)=f(x+1)-f(x)=f'(c_x)$.



      If $xrightarrow infty$, then $c_x rightarrow infty$. Therefore,
      $lim_xrightarrow inftyg(x)=lim_xrightarrowinftyf'(c_x)=lim_xrightarrowinftyf'(x)=0$






      share|cite|improve this answer









      $endgroup$

















        1












        $begingroup$

        By the mean value theorem for differentiable functions, for $x>0$ there exists $x< c_x< x+1$ such that $g(x)=f(x+1)-f(x)=f'(c_x)$.



        If $xrightarrow infty$, then $c_x rightarrow infty$. Therefore,
        $lim_xrightarrow inftyg(x)=lim_xrightarrowinftyf'(c_x)=lim_xrightarrowinftyf'(x)=0$






        share|cite|improve this answer









        $endgroup$















          1












          1








          1





          $begingroup$

          By the mean value theorem for differentiable functions, for $x>0$ there exists $x< c_x< x+1$ such that $g(x)=f(x+1)-f(x)=f'(c_x)$.



          If $xrightarrow infty$, then $c_x rightarrow infty$. Therefore,
          $lim_xrightarrow inftyg(x)=lim_xrightarrowinftyf'(c_x)=lim_xrightarrowinftyf'(x)=0$






          share|cite|improve this answer









          $endgroup$



          By the mean value theorem for differentiable functions, for $x>0$ there exists $x< c_x< x+1$ such that $g(x)=f(x+1)-f(x)=f'(c_x)$.



          If $xrightarrow infty$, then $c_x rightarrow infty$. Therefore,
          $lim_xrightarrow inftyg(x)=lim_xrightarrowinftyf'(c_x)=lim_xrightarrowinftyf'(x)=0$







          share|cite|improve this answer












          share|cite|improve this answer



          share|cite|improve this answer










          answered Mar 27 at 21:37









          folouer of kaklasfolouer of kaklas

          338110




          338110





















              1












              $begingroup$

              MVT:



              $g(x)=f(x+1)-f(x)= f'(t_x) cdot 1$; where $x <t_x <x+1$.



              We have $lim_x rightarrow infty f'(x)=0:$



              Let $epsilon >0$ be given.



              There is a $M >0$, real, s.t. for $x >M$:



              $|f'(x)| < epsilon.$



              For $x >M$:



              Since $g(x)=f'(t_x)$ , where $x <t_x,$ we have



              $|g(x)| =|f'(t_x)| < epsilon,$ i .e



              $lim_x rightarrow inftyg(x)=0.$






              share|cite|improve this answer











              $endgroup$

















                1












                $begingroup$

                MVT:



                $g(x)=f(x+1)-f(x)= f'(t_x) cdot 1$; where $x <t_x <x+1$.



                We have $lim_x rightarrow infty f'(x)=0:$



                Let $epsilon >0$ be given.



                There is a $M >0$, real, s.t. for $x >M$:



                $|f'(x)| < epsilon.$



                For $x >M$:



                Since $g(x)=f'(t_x)$ , where $x <t_x,$ we have



                $|g(x)| =|f'(t_x)| < epsilon,$ i .e



                $lim_x rightarrow inftyg(x)=0.$






                share|cite|improve this answer











                $endgroup$















                  1












                  1








                  1





                  $begingroup$

                  MVT:



                  $g(x)=f(x+1)-f(x)= f'(t_x) cdot 1$; where $x <t_x <x+1$.



                  We have $lim_x rightarrow infty f'(x)=0:$



                  Let $epsilon >0$ be given.



                  There is a $M >0$, real, s.t. for $x >M$:



                  $|f'(x)| < epsilon.$



                  For $x >M$:



                  Since $g(x)=f'(t_x)$ , where $x <t_x,$ we have



                  $|g(x)| =|f'(t_x)| < epsilon,$ i .e



                  $lim_x rightarrow inftyg(x)=0.$






                  share|cite|improve this answer











                  $endgroup$



                  MVT:



                  $g(x)=f(x+1)-f(x)= f'(t_x) cdot 1$; where $x <t_x <x+1$.



                  We have $lim_x rightarrow infty f'(x)=0:$



                  Let $epsilon >0$ be given.



                  There is a $M >0$, real, s.t. for $x >M$:



                  $|f'(x)| < epsilon.$



                  For $x >M$:



                  Since $g(x)=f'(t_x)$ , where $x <t_x,$ we have



                  $|g(x)| =|f'(t_x)| < epsilon,$ i .e



                  $lim_x rightarrow inftyg(x)=0.$







                  share|cite|improve this answer














                  share|cite|improve this answer



                  share|cite|improve this answer








                  edited Mar 28 at 4:29

























                  answered Mar 27 at 22:27









                  Peter SzilasPeter Szilas

                  12k2822




                  12k2822





















                      0












                      $begingroup$

                      The proof is simple:



                      $lim_x rightarrow infty g(x)=lim_xrightarrow infty f(x+1)-lim_xrightarrow infty f(x)$



                      For large $x$, $x+1 approx x$, so, by the previous result,



                      $lim_x rightarrow infty g(x)=0$






                      share|cite|improve this answer









                      $endgroup$












                      • $begingroup$
                        One has to also prove that $lim_xrightarrow infty f(x)$ exists!
                        $endgroup$
                        – folouer of kaklas
                        Mar 27 at 21:30
















                      0












                      $begingroup$

                      The proof is simple:



                      $lim_x rightarrow infty g(x)=lim_xrightarrow infty f(x+1)-lim_xrightarrow infty f(x)$



                      For large $x$, $x+1 approx x$, so, by the previous result,



                      $lim_x rightarrow infty g(x)=0$






                      share|cite|improve this answer









                      $endgroup$












                      • $begingroup$
                        One has to also prove that $lim_xrightarrow infty f(x)$ exists!
                        $endgroup$
                        – folouer of kaklas
                        Mar 27 at 21:30














                      0












                      0








                      0





                      $begingroup$

                      The proof is simple:



                      $lim_x rightarrow infty g(x)=lim_xrightarrow infty f(x+1)-lim_xrightarrow infty f(x)$



                      For large $x$, $x+1 approx x$, so, by the previous result,



                      $lim_x rightarrow infty g(x)=0$






                      share|cite|improve this answer









                      $endgroup$



                      The proof is simple:



                      $lim_x rightarrow infty g(x)=lim_xrightarrow infty f(x+1)-lim_xrightarrow infty f(x)$



                      For large $x$, $x+1 approx x$, so, by the previous result,



                      $lim_x rightarrow infty g(x)=0$







                      share|cite|improve this answer












                      share|cite|improve this answer



                      share|cite|improve this answer










                      answered Mar 27 at 21:24









                      officialnoriaofficialnoria

                      112




                      112











                      • $begingroup$
                        One has to also prove that $lim_xrightarrow infty f(x)$ exists!
                        $endgroup$
                        – folouer of kaklas
                        Mar 27 at 21:30

















                      • $begingroup$
                        One has to also prove that $lim_xrightarrow infty f(x)$ exists!
                        $endgroup$
                        – folouer of kaklas
                        Mar 27 at 21:30
















                      $begingroup$
                      One has to also prove that $lim_xrightarrow infty f(x)$ exists!
                      $endgroup$
                      – folouer of kaklas
                      Mar 27 at 21:30





                      $begingroup$
                      One has to also prove that $lim_xrightarrow infty f(x)$ exists!
                      $endgroup$
                      – folouer of kaklas
                      Mar 27 at 21:30


















                      draft saved

                      draft discarded
















































                      Thanks for contributing an answer to Mathematics Stack Exchange!


                      • Please be sure to answer the question. Provide details and share your research!

                      But avoid


                      • Asking for help, clarification, or responding to other answers.

                      • Making statements based on opinion; back them up with references or personal experience.

                      Use MathJax to format equations. MathJax reference.


                      To learn more, see our tips on writing great answers.




                      draft saved


                      draft discarded














                      StackExchange.ready(
                      function ()
                      StackExchange.openid.initPostLogin('.new-post-login', 'https%3a%2f%2fmath.stackexchange.com%2fquestions%2f3165068%2fsuppose-f-is-defined-and-differentiable-proof%23new-answer', 'question_page');

                      );

                      Post as a guest















                      Required, but never shown





















































                      Required, but never shown














                      Required, but never shown












                      Required, but never shown







                      Required, but never shown

































                      Required, but never shown














                      Required, but never shown












                      Required, but never shown







                      Required, but never shown







                      Popular posts from this blog

                      Lowndes Grove History Architecture References Navigation menu32°48′6″N 79°57′58″W / 32.80167°N 79.96611°W / 32.80167; -79.9661132°48′6″N 79°57′58″W / 32.80167°N 79.96611°W / 32.80167; -79.9661178002500"National Register Information System"Historic houses of South Carolina"Lowndes Grove""+32° 48' 6.00", −79° 57' 58.00""Lowndes Grove, Charleston County (260 St. Margaret St., Charleston)""Lowndes Grove"The Charleston ExpositionIt Happened in South Carolina"Lowndes Grove (House), Saint Margaret Street & Sixth Avenue, Charleston, Charleston County, SC(Photographs)"Plantations of the Carolina Low Countrye

                      How should I support this large drywall patch? Planned maintenance scheduled April 23, 2019 at 00:00UTC (8:00pm US/Eastern) Announcing the arrival of Valued Associate #679: Cesar Manara Unicorn Meta Zoo #1: Why another podcast?How do I cover large gaps in drywall?How do I keep drywall around a patch from crumbling?Can I glue a second layer of drywall?How to patch long strip on drywall?Large drywall patch: how to avoid bulging seams?Drywall Mesh Patch vs. Bulge? To remove or not to remove?How to fix this drywall job?Prep drywall before backsplashWhat's the best way to fix this horrible drywall patch job?Drywall patching using 3M Patch Plus Primer

                      random experiment with two different functions on unit interval Announcing the arrival of Valued Associate #679: Cesar Manara Planned maintenance scheduled April 23, 2019 at 00:00UTC (8:00pm US/Eastern)Random variable and probability space notionsRandom Walk with EdgesFinding functions where the increase over a random interval is Poisson distributedNumber of days until dayCan an observed event in fact be of zero probability?Unit random processmodels of coins and uniform distributionHow to get the number of successes given $n$ trials , probability $P$ and a random variable $X$Absorbing Markov chain in a computer. Is “almost every” turned into always convergence in computer executions?Stopped random walk is not uniformly integrable